pls help will give brainly

Pls Help Will Give Brainly

Answers

Answer 1

Answer:

-8

Step-by-step explanation:

Since x = -18, we use the top formula

f(-18) = (x-6)/3

        = (-18-6)/3

        = -24/3

       = -8


Related Questions

Please help pleaseeee

Answers

Answer:

False

Step-by-step explanation:

When you compare the dots to the numbers, you can see that the 5s and 8s don't match.

The dot plot has 6 5s, but the data has 5.

The dot plot has 2 8s, but the data has 1.

I hope this helps!

pls ❤ and give brainliest pls

Organize the data set:

8,7,5,5,4,2,3,3,4,6,5,4,5,5

2,3,3,4,4,4,5,5,5,5,5,6,7,8

False

What is the sum of -6 and 9?

Answers

Sum means add.

Start at -6 and add 9

-6 + 9 = 3

Answer: 3

[tex]\huge\text{Hey there!}[/tex]

[tex]\huge\text{What is the sum of -6 \& 9?}[/tex]

[tex]\huge\textsf{The word \underline{sum} means to \bf add/addition }[/tex]

[tex]\huge\text{The EQUATION: -6 + 9 = [blank]}[/tex]

[tex]\huge\bullet\huge\textsf{To find out your result you have to start}\\\huge\textsf{at -6 \& go UP 9 spaces to the right}\\\huge\textsf{since you are ADDING \& NOT}\\\huge\textsf{SUBTRACTING.}\\\\\huge\bullet\huge\textsf{If you are subtracting you would do the}\\\huge\textsf{opposite \& go down to the left}[/tex]

[tex]\huge\boxed{\star}\huge\text{ Without further ado, we have reached}\\\huge\text{our result to your question is \boxed{\bf 3}}[/tex]

[tex]\boxed{\boxed{\huge\text{Answer: \bf 3}}}\huge\checkmark[/tex]

[tex]\huge\textsf{Good luck on your assignment \& enjoy your day!}[/tex]

~[tex]\huge\boxed{\frak{Amphitrite1040:)}}[/tex]

Help please, area of a triangle

Answers

Answer:

84 ft^2.

Step-by-step explanation:

Use Pythagoras to find the value of h:

25^2 = h^2 + 7^2

h^2 = (25-7)(25+7)

h^2 = 576

h = √576 = 24

Area  = 1/2 * 7 * 24

= 84.


16. Mrs. Acosta wants to buy 2 flag pins
for each of the 168 band members for
the Fourth of July Parade. Pins cost
$0.09 each. Which is the best estimate
of the cost of the pins?


Answers

Because we are looking for an estimate, we can round 168 people to 170. Then, 2 pins for about 170 people would be 340 pins (simply multiple 170 x 2). Then we can round 9 cents to 10 cents, or .1 dollars. 340 pins times .1 is just 34 dollars (move the decimal once to the left).

Answer: About 34 dollars

14 POINTER IF YOU HIT THE BULLSEYE!!!!! HELP!
Billy, Bob, and Joe found a question in their test that said this and they NEED HELP!!!!! with flying promises to give brainliest they now wait.......

Answers

Answer:

mean would decrease

Step-by-step explanation:

Find the sum and express it in simplest form.
(6u - 7c - 6) + (-2u + 4c) ​

Answers

Answer:

4u - 3c - 6

Step-by-step explanation:

Remove the parentheses and combine like factors.

(6u - 7c - 6) + (-2u + 4c)

6u - 7c - 6 - 2u + 4c

4u - 3c - 6

8c + 2(6 - 3c) - 7
How do I simplify this pls help

Answers

Answer:

2c+5

Step-by-step explanation:

Answer:

7c

Step-by-step explanation:

You do PEMDAS.

6-3c is 3c.

so itd be 8c + 2(3c) - 7

then id multiply 3c with 2 so its be6c.

then id add 6c with 8c, and its be 14c. then i subtract it with 7 and get 7c.

Please please please please help

Answers

Answer:

[tex]10 {x}^{2} + ( - 18)[/tex]

Step-by-step explanation:

Inserting fx into gx we get

[tex]5(2 {x}^{2} - 5) + 7[/tex]

Which then becomes

[tex]10 {x}^{2} - 25 + 7[/tex]

And finally the answer is

[tex]10 {x}^{2} - 18[/tex]

Please Help!!!!!!!
Janet is going to draw a marble from the bag shown below, replace it, and then draw another marble. What is the
probability that Janet will get a blue or yellow marble on the first draw and a pink on the second draw?
(3 green, 4 yellow, 5 blue, 8 pink)
7/50
9/50
17/40

Answers

(Answer:

P(Bl or Yl) = 45%

(Pk on 2nd draw) = 18%

Step-by-step explanation:

There are a total of 20 marbles (3 + 4 + 5 + 8) in the bag. Since each marble is replaced once drawn, there is no change in the total count from one draw to another (unconditional probability).

So

P(Bl or Yl) = (5 + 4)/20 = 9/20 = .45 = 45%

Then

P(Pk on 2nd draw) = P(1st draw) * P(2nd draw)

     = 9/20 * 8/20 = 72/400 = 9/50 = .18 = 18%

PLS HELP ME
Given that the universal set.
U = {X : 30 ≤ X ≤ 40. X is an even number}.
P = {X : X is a multiple of 4}
and Q = {X : X is a number such that the sum of its digits is an odd number}.
find n(P∩Q')

A. 1
B. 2
C. 3
D. None of the above​

Answers

Given that

U = {X : 30 ≤ X ≤ 40. X is an even number}.

so Mathematically

U = {30,32,34,36,38 40}.

P = {X : X is a multiple of 4}

P = { 4,8,12,16,20,24,28,32,36,40,44,........}

and Q = {X : X is a number such that the sum of its digits is an odd number}.

Q = {12,14,16,18,21,23,25,27,29,30,32,34,36,38,41,43,45,..........}.

(Q')= U-Q  ={40}

(P∩Q')= {40}

n(P∩Q')= 1

(Q') is Q complement which means a set containing those elements of the universal set which are not element of the set Q.

(P∩Q') is a set containing those elements of set P and Q complement which are common to both sets.

n(P∩Q') denotes the number of elements in a set containing those elements of set P and Q complement which are common to both sets which  has only 1 element which is 40.

So the best answer is part A = 1.

https://brainly.com/question/11439790

https://brainly.com/question/13729460

4+2p=10 (3/4p-2) solve for p

Answers

Answer:

p = 48/11 or 4.36

Step-by-step explanation:

4 + 2p = 10(3/4p - 2)

distribute the 10 on the right side of the equation

4 + 2p = (15/2p - 20)

multiply both sides by 2

8 + 4p = 15p - 40

move the terms

48 = 11p

p = 48/11

(sorry if this question is already answered, brainly is glitching out for me)

Answer:

p=6

I got it right on Kahn Academy

use an algebraic equation to find the measures of the two angles described below. begin by letting x represent the degree measure of the angles supplement.

the measure of the angle is twenty-nine times greater than its supplement

Answers

X+29x= 180
30x= 180
One angle is 6 the other is 174

BD is paralell to XY, what is the value of y?
A. 125
B. 85
C. 65
D. 105​

Answers

Answer:

options b is correct

Does the answer help you?

Answer:

B. [tex]85[/tex]

Explanation:

This is a simple test of your knowledge about this sort of problem. When you get 2 parallel lines intersected the way you see in the picture, the lower right angle of the first intersection is equal to the upper left angle of the first line as well as the upper left and lower right angles of the second line so with that knowledge you can immediately identify that since the question gives you the measure of an angle equal to the angle you need to identify; you don't even need to do any math to find the answer for this question.

PLS HELP! Given the angles in the diagram below, what is m<2

Answers

Your question answer is d 98 is right.

please help me you will recieve 5 stars IF RIGHT ANSWER !

Answers

Answer:

[tex]\huge\boxed{\frac{7}{8}}[/tex]

Step-by-step explanation:

[tex](\frac{49 }{64})^{1/2}[/tex]

=> [tex](\frac{7^2}{8^2} )^{1/2}[/tex]

=> [tex]\frac{7^{2*1/2}}{8^{2*1/2}}[/tex]

=> [tex]\frac{7}{8}[/tex]

Answer:

Below

Step-by-step explanation:

You should now that:

● (m/n)^(1/2) = √(m/n)

So:

● (49/64)^(1/2) = √(m/n)

You shoukd now also that:

● √(m/n) = √m / √n

So:

● √(49/64) = √49/√64

Notice that 64 = 8^2 and 49 = 7^2

● √49 / √64 = √(7^2)/√(8^2) = 7/8

So the answer is 7/8

Simplify this Algebraic equation with steps
-6q +7 -2 +4q

Answers

Answer:

-2q+5

Step-by-step explanation:

-6q +7 -2 +4q

Combine like terms

-6q+4q+7-2

-2q+5

Answer:

= solution,

= -6q + 7 - 2 +4q

= -6q + 4q +7 -2

= - 2q + 5

✌✌✌✌

Type the correct answer in the box. Use numerals instead of words. If necessary, use / for the fraction bar.

Answers

Answer:

2/5

Step-by-step explanation:

First for the sum:

3/5 + 1/5 i + 4/5 - 2/5 i = 7/5 - 1/5 i

Now for the difference

9/5 - 1/5 i - (7/5 - 1/5 i)

= 9/5 - 1/5 i - 7/5 + 1/5 i

= 2/5 , which is the answer :)

The answer is  2/5.

To find the fraction bar in the box.

What is fraction?

A fraction is a part of a whole. In arithmetic, the number is expressed as a quotient, in which the numerator is divided by the denominator. In a simple fraction, both are integers. A complex fraction has a fraction in the numerator or denominator. In a proper fraction, the numerator is less than the denominator.

Given that:

First for the sum:

3/5 + 1/5 i + 4/5 - 2/5 i = 7/5 - 1/5 i

Now substract the sum,

9/5 - 1/5 i - (7/5 - 1/5 i)

= 9/5 - 1/5 i - 7/5 + 1/5 i

= 2/5

So, the answer is  2/5.

Learn more about fraction here:

https://brainly.com/question/17154057

#SPJ2

expand the decimal number 453682​

Answers

0.453682

= 0.4+0.05+0.003+0.0006+0.00008+0.000002

453682.0

= 400000+50000+3000+600+80+2+0.0

Must click thanks and mark brainliest

ans this question......​

Answers

Step-by-step explanation:

Universal Set(U)={2,3,4...16}

M={2,4,6,8..16}[Even numbers between 2-16]

N={3,5,7...15}[Odd numbers between 2-16]

(PUM)'=U-(PUM)={2,3,4...16}-[{2,4,6,8..16}U{3,5,7...15}]={}

For shading, Shade the area outside the two circles but inside the square.

Cole deposits $500 into his savings account. The bank pays him 2% interest on the amount he has in his account. What expression would you

use to find the total Cole has in his account?

Answers

Answer:

500*2%=10

500+10=510

Step-by-step explanation:

Sondra receives an allowance of $10 per week, plus an additional $5 for each chore she completes. Which graph represents Sondra's earnings?

Answers

Answer: I know this is a bit late but for anyone looking the answer to this question, i got it right here :D.

Step-by-step explanation: I guessed and got it correct.

The graph representing Sondra's weekly earnings is plotted and attached.

What is a expression? What is a mathematical equation? What is Equation modelling ?

A mathematical expression is made up of terms (constants and variables) separated by mathematical operators. A mathematical equation is used to equate two expressions. Equation modelling is the process of writing a mathematical verbal expression in the form of a mathematical expression for correct analysis, observations and results of the given problem.

Given is Sondra receives an allowance of $10 per week, plus an additional $5 for each chore she completes.

Assume that her weekly earnings is $y. Assume she does [x] extra chores in a week. Then, we can model her earnings by the equation -

y = 5x + 10.

Refer to the graph attached.

Therefore, the graph representing Sondra's weekly earnings is plotted and attached.

To solve more questions on Equations, Equation Modelling and Expressions visit the link below -

brainly.com/question/14441381

#SPJ2

evaluate the following 3^2 divided by (2+1)

Answers

Answer:

God bless you

i don't have nice handwriting

find the area of a sector with a central angle of 150 and a diameter of 7.8 cm

Answers

Answer:

19.9 cm²

Step-by-step explanation:

The following data were obtained from the question:

Angle at the centre (θ) = 150°

Diameter (d) = 7.8 cm

Area of sector =.?

Next, we shall determine the radius.

This can be obtained as illustrated below:

Radius (r) = Diameter (d) /2

r = d/2

Diameter (d) = 7.8 cm

Radius (r) =.?

r = d/2

r = 7.8/2

r = 3.9 cm

Finally, we shall determine the area of the sector as follow:

Angle at the centre (θ) = 150°

Radius (r) = 3.9 cm

Pi (π) = 3.14

Area of sector (A) =.?

A = θ/360 × πr²

A = 150/360 × 3.14 × 3.9²

A = 15/36 × 3.14 × 15.21

A = 19.9 cm²

Therefore, the area of the sector is 19.9 cm²

Special right triangles

Answers

Answer: please find the attached files

Step-by-step explanation:

A unit circle formula and special triangle of 45, 30 and 60 degrees can be used to solve the problem.

Please find the attached files for the solution

Journal Write a subtraction problem involving
numbers. At least one decimal must include hundredths. Explain how to solve
your problem.
Please help I will give you Brainlyest

Answers

Answer:

Problem: 5.24 - 4.93

Step-by-step explanation:

You subtract just like a normal Addition problem however you MUST make sure the decimal point line up,...

[tex]5.24\\4.93\\[/tex]

1st: subtract the 3 from the 4 [0.01]

2nd: subtract the 9 from the 2 (make the 2 a 12 (by carrying a one from the 5 (which then becomes a 4))) [0.30]

3rd: Finally subtract the 4 from the 4,... 0

Answer = 0.31,...

A more summed up answer: Subtracting (and adding for that matter) decimals happens exactly like normal. However you MUST MAKE SURE that your decimal points are lined up,... if you are subtracting a 0.09 from a 4.5 and the 4.5 doesn't have a hundredths place just remember that you can add a 0 on the end (4.50) without chancing it's value. (and then just subtract like normal)

WHERE ARE THE EXPERTS AND ACE!!!!!!! I NEED HELP PLS SHARE YO SMARTNESS!!!!! WILL GIVE BRAINLIEST AND RATE AND VOTE!!! EASY IM JUST NOT SMART

Answers

Answer:

7.2

Step-by-step explanation:

Apply Law of Cosines, given that b is 13.5, c is 8.9, and Angle a is 29.

[tex] {a}^{2} = {b}^{2} + {c}^{2} - 2bc \cos(a) [/tex]

[tex] {a}^{2} = {13.5}^{2} + {8.9}^{2} - 2(13.5)(8.9) \times \cos(29) [/tex]

[tex] {a}^{2} = 182.25 + 79.21 - 240.3 \times 0.8746[/tex]

[tex] {a}^{2} = 182.25 + 79.21 - 210.1711[/tex]

[tex]a = 7.2[/tex]

Answer:

[tex]a\approx7.2[/tex]

Step-by-step explanation:

The given problem presents one with the following information:

Triangle ABCSide (b) has a measure of (13.5)Side (c) has a measure of (8.9)Angle (A) has a measure of (29)

To solve this problem, one can use the law of cosines. The law of cosines is a property that can apply to any triangle. This property comes in the form of a formula, which is as follows:

[tex]a^2=\sqrt{b^2+c^2-2bc(cos(A))}[/tex]

Where (a), (b), and (c) are sides of the triangle, and (<A) is the angle opposite the side (a).

Substitute the given information into the formula and solve for the unknown side (a):

[tex]a=\sqrt{b^2+c^2-2bc(cos(A))}[/tex]

[tex]a=\sqrt{(13.5)^2+(8.9)^2-2(13.5)(8.9)(cos(29))}[/tex]

Simplify,

[tex]a=\sqrt{(13.5)^2+(8.9)^2-2(13.5)(8.9)(cos(29))}[/tex]

[tex]a=\sqrt{182.25+79.21-240.3(cos(29))}[/tex]

[tex]a=\sqrt{261.46-240.3(cos(29))}[/tex]

[tex]a=\sqrt{261.46-210.1711}[/tex]

[tex]a=\sqrt{51.2889}[/tex]

[tex]a\approx7.2[/tex]

Please answer this question now

Answers

Answer:

320 square inches

Step-by-step explanation:

4 * 1/2(8)(16) + 8*8 = 320

Answer:

320 sq. in.

Step-by-step explanation:

The formula for finding the area of a triangle is:

[tex]\frac{hb}{2}[/tex] (basically multiplying the height and the base and then dividing by 2)

Since there are 4 triangles, we can multiply the area of 1 triangle by 4 (64 times 4 is 256).

Then, on the bottom we have a (8 times 8) square (64).

Triangles: 256

Square: 64

256 + 64 = 320 sq. in!

Hope that helps and maybe earns a brainliest!

Have a great day!

Rational and irrational numbers are both imaginary numbers.
True
False
help please! thank you :)

Answers

Answer:

The correct option is;

False

Step-by-step explanation:

An imaginary number is also known as a imaginary part of a complex number is a real number that has a factor of √(-1)

Rational numbers are numbers that can be put in the form of the ratio of two integers (real numbers), forming a simple fraction such as 1/2, or 3/7

Irrational numbers are the subset of real numbers that cannot be expressed as a ratio of two numbers such as π, √2, Eulers number, e, the golden ratio, φ

Therefore, rational numbers and irrational numbers are real numbers not imaginary numbers.

i need help. Can u help me solve for x?

Answers

Answer:

[tex] x = \sqrt {40}[/tex]

Step-by-step explanation:

Given is an isosceles triangle, dotted line is the bisector of top angle which is also perpendicular bisector of the base of the triangle. Hence, by Pythagoras theorem:

[tex] {x}^{2} = {6}^{2} + ({ \frac{4}{2} })^{2} \\ = 36 + 4 \\ = 40 \\ \therefore \: x = \sqrt{40} \\ [/tex]

Answer:

D. x = sqrt(52).

Step-by-step explanation:

Since the line measuring 6 units bisects the top angle, there are two right angles. We can use the Pythagorean Theorem to solve for x.

a^2 + b^2 = x^2

4^2 + 6^2 = x^2

16 + 36 = x^2

52 = x^2

x = sqrt(52)

x = sqrt(2 * 2 * 13)

x = 2sqrt(13)

x = 7.211102551.

Hope this helps!

Can someone answer this please? Okay so it says that something is made 3 times than the other item. The other item uses 13 beads. So, what is 13 times 3?

Answers

Answer:

13x3= 39

Step-by-step explanation:

10x3=30

3x3=9

30+9=39

Hope it helps!

If you multiply them it would be 39. Hope this helps
Other Questions
Espinoza Company is a wholesale distributor that uses activity-based costing for all of its overhead costs. The company has provided the following data concerning its annual overhead costs and its activity based costing system: Overhead costs: Wages and salaries 220,000Other expenses 150,000Total $510,000Distribution of resource consumption: Filling Orders Activity Cost Pools Customer Support Other TotalWages and salaries 35% 55% 10% 100%Other expenses 35% 50% 15% 100%The "Other" activity cost pool consists of the costs of idle capacity and organization-sustaining costs. The activity measures for the activity cost pools for the year are as follows:Activity Cost Pool ActivityFilling orders 3,500 ordersCustomer support 15 customers.What would be the overall activity rate for the filling orders activity cost pool? Assume MIX Inc. has sales volume of $1,198,000 for two products with May sales and contribution margin ratios as follows:Product A: Sales $466,000; Contribution Margin Ratio 30%Product B: Sales $732,000; Contribution Margin Ratio 60%Required: Assume MIX's fixed expenses are $302,000. Calculate the May total contribution margin, operating income, average contribution margin ratio, and breakeven sales volume. (Round "Average contribution margin ratio" answer to 2 decimal places. Round up "Breakeven sales volume" answer to nearest whole dollar.) Total contribution margin Operating income Average contribution margin ratio Breakeven sales volume ____is associated with deamination of protein Help ASAP please In your own words what is the definition and put them In a sentence of synecdoche, personification, Allusion, Onomatopoeia, and Alliteration (NO COPY AND PASTE!!!)Words for sentence/ definition in your own wordsAllusion Alliteration Onomatopoeia Synecdoche Personification If a sample of aluminum with a density of 2.70 g/cm^3 displaces 36.0 mL of water when placed in a beaker, what is its mass? 13.3 g 97.2 g 0.075 g Will give brainliest to correct answer Please answer ASAP, the assignments in due really soon :) 30 Points!!! The image below shows the lac operon, which breaks down lactose in bacteria. Which statement correctly describes the expression of this gene? A bookstore decides to divide its space into three sections: nonfiction books, novels, and stationery. The bookstore wants to devote 1/6 of its space to stationery. If the total area of the bookstore is 288 square feet, and the stationery section will be 12 feet long, how wide will the stationery section be? 3 ejemplos fundamentados de la diversidad sexual. Which statement best compares the main characters in "The Prodigal Son" and "The Glass of Milk"? Pamela is 8 years older that Jiri. The sum of their age is 102. What is Jiri's age? 88 feet/second = 60 miles/hour. How many feet per second is 1 mile/hour? Type the correct answer in the box. Spell all words correctly.Which lawmaking body forms a part of the legislative branch?The legislative branch of the US government comprises the _______ , which is the greatest lawmaking authority in the country. If oxygen causes fire to burn, why didn't the world explode when fire burns at a particular place...since oxygen is everywhere...? Explain... In a transaction for a good valued at $550 by a buyer and $500 by a seller, what percentage sales tax (assume tax on $500) would result in an unconsummated transaction write the four names of disease,causative agent,disease spread mechanism, precautions.give the correct answer. What is the midline equation of the function h(x) = -4 cos(5x - 9) - 7? a mens clothing sore sold out of $50 jackets and $30 jackets for a total of $2360 if the store sold 12 more$30 jackets than$50 jackets how many$50 jackets were sold plz help me. how many solutions trigonometric identities